Đến nội dung

duongvanhehe

duongvanhehe

Đăng ký: 01-07-2012
Offline Đăng nhập: 04-01-2015 - 18:49
***--

#369596 $\frac{a}{b}+\frac{b}{c...

Gửi bởi duongvanhehe trong 15-11-2012 - 13:23

Bài 1: Cho $a,b,c$ là các số thực dương. CMR:
$\frac{a}{b}+\frac{b}{c}+\frac{c}{a}+2\sqrt{2}.\frac{ab+bc+ca}{a^2+b^2+c^2}\geq (1+\sqrt{2})^2$

Bai2:
$\left ( 1+\frac{4a^2}{(b+c)^2} \right ).\left ( 1+\frac{4b^2}{(c+a)^2} \right ).\left ( 1+\frac{4c^2}{(a+b)^2} \right )\geq 8$


#369591 $\frac{a^2}{b^2+c^2}+\frac{b^2}...

Gửi bởi duongvanhehe trong 15-11-2012 - 12:37

Bài toán:
Cho $a,b,c$ là các số thực không âm. Chứng minh rằng:
$\frac{a^2}{b^2+c^2}+\frac{b^2}{c^2+a^2}+\frac{c^2}{a^2+b^2}+6\geq 4.\frac{a^2+b^2+c^2}{ab+bc+ca}$


#369495 $2.\frac{(a+b+c)^3-11abc}{(a+b)^2(b+c)^2(c+a)^2...

Gửi bởi duongvanhehe trong 14-11-2012 - 21:05

Bài toán:Cho $a,b,c\geq 0$ . CMR:
$2.\frac{(a+b+c)^3-11abc}{(a+b)^2(b+c)^2(c+a)^2}\geq \frac{1}{a^2b+b^2c+c^2a+abc}+\frac{1}{ab^2+bc^2+ca^2+abc}$


#369345 $\frac{(a+b+c)^3}{a^2b+b^2c+c^2a+3abc}\geq...

Gửi bởi duongvanhehe trong 13-11-2012 - 23:21

với mọi a,b,c>0,có BĐT này không mọi người(giúp em với)
$\frac{(a+b+c)^3}{a^2b+b^2c+c^2a+3abc}\geq \frac{9}{2}$

Có chứ:
$a^2b+b^2c+c^2a+abc\leq \frac{4}{27}(a+b+c)^3$ ( http://diendantoanho...c2aabbccaleq-9/ )
và $2abc\leq \frac{2}{27}(a+b+c)^3$
nên $\frac{(a+b+c)^3}{a^2b+b^2c+c^2a+3abc}\geq \frac{1}{\frac{4}{27}+\frac{2}{27}}=\frac{9}{2}$


#369324 $$a^3+b^3+c^3+3abc\geq \frac{(a^2b+b^2c+c^2a)^2...

Gửi bởi duongvanhehe trong 13-11-2012 - 22:35

Bài toán 1.
Ch0 các số thực không âm $a,b,c$ thỏa $a+2b+3c=4$.Chứng minh rằng:
$$(a^2b+b^2c+c^2a+abc)(ab^2+bc^2+ca^2+abc)\leq 8$$
Bài toán 2.
Chứng minh $\forall$ số thực không âm $a,b,c$ ta luôn có bất đẳng thức:
$$a^3+b^3+c^3+3abc\geq \frac{(a^2b+b^2c+c^2a)^2}{ab^2+bc^2+ca^2}+\frac{(ab^2+bc^2+ca^2)^2}{a^2b+b^2c+c^2a}$$

Bài 1:
Không giảm tính tổng quát giả sử $c$ là số lớn nhất trong $a,b,c$.Ta có:
$a^2b+b^2c+c^2a+abc\leq a^2b+c^2a+abc+(abc+bc^2+ca^2+ac^2+c^3)=(a+c)^2(b+c)$
Tương tự $ab^2+bc^2+ca^2+abc\leq (a+c)(b+c)^2$
$\Rightarrow (a^2b+b^2c+c^2a+abc)(ab^2+bc^2+ca^2+abc)\leq (a+c)^3(b+c)^3$
$=\frac{1}{8}\left ( (a+c)(2b+2c) \right )^3\leq \frac{1}{8}.\left [ \left ( \frac{a+c+2b+2c}{2} \right )^2 \right ]^3=8$
Đẳng thức xảy ra khi $a=2,b=1,c=0$

Bài 2:
Khai triển BĐT này ta được BĐT tương đương :
$abc\left ( 12a^2b^2c^2+3(a^3b^3+b^3c^3+c^3a^3)+(a^3+b^3+c^3)^2+6abc(a^3+b^3+c^3)-6(a^2+bc)(b^2+ca)(c^2+ab) \right )\geq 0$
Điều này hiển nhiên đúng vì :
$12a^2b^2c^2+3(a^3b^3+b^3c^3+c^3a^3)+(a^3+b^3+c^3)^2+6abc(a^3+b^3+c^3)\geq 12a^2b^2c^2+6(a^3b^3+b^3c^3+c^3a^3)+6abc(a^3+b^3+c^3)= 6(a^2+bc)(b^2+ca)(c^2+ab)$
Đẳng thức xảy ra khi $a=b=c$ hoặc $abc=0$
:lol: bài này có thể xem thêm 1 cách rất hay trong BĐT và Những lời giải hay (trang 148)
---------------
chết thật,sai lầm nghiêm trọng :(


#369215 $$10+ab^2+bc^2+ca^2\geq \frac{7}{8}.(...

Gửi bởi duongvanhehe trong 13-11-2012 - 19:35

Bài toán 1.
Ch0 các số thực không âm $a,b,c$.Chứng minh rằng:
$$\frac{(a+b)^2(a+c)^2}{(b^2-c^2)^2}+\frac{(a+b)^2(b+c)^2}{(a^2-c^2)^2}+\frac{(b+c)^2(a+c)^2}{(a^2-b^2)^2}\geq 2$$

Ta có:
$\sum \frac{(a+b)(a+c)}{b^2-c^2}.\frac{(b+a)(b+c)}{c^2-a^2}=\sum \frac{(a+b)^2}{(b-c)(c-a)}$
$=\frac{\sum (a+b)^2(a-b)}{(a-b)(b-c)(c-a)}=\frac{a^2b+b^2c+c^2a-ab^2-bc^2-ca^2}{(a-b)(b-c)(c-a)}=-1$
$\Rightarrow \sum \left ( \frac{(a+b)(a+c)}{b^2-c^2} \right )^{2}\geq -2\left ( \frac{(a+b)(a+c)}{b^2-c^2}.\frac{(a+b)(b+c)}{c^2-a^2} \right )=2$


Bài toán 2.
Chứng minh với mọi số thực không âm $a,b,c$ có tổng bằng 5 ta luôn có:
$$10+ab^2+bc^2+ca^2\geq \frac{7}{8}.(a^2b+b^2c+c^2a)$$

BĐT$\Leftrightarrow 160+16(ab^2+bc^2+ca^2)\geq 14(a^2b+b^2c+c^2a)$
$\Leftrightarrow 160+ab(a+b)+bc(b+c)+ca(c+a)\geq 15(a-b)(b-c)(a-c)$
Giả sử $c=min{a,b,c}$ Ta có:
$160+ab(a+b)+bc(b+c)+ca(c+a)\geq \frac{160}{25}(a-c+b-c)^3+(a-c)(b-c)(a+b-2c)$
Tức là giờ ta chỉ cần chứng minh BĐT khi $c=0$ .Tương đương
$\frac{160}{25}(a+b)^3+16ab^2\geq 14a^2b\Leftrightarrow (a-4b)^2(16a+b)\geq 0$ (TRUE)
Đẳng thức xảy ra khi $(a,b,c)$ là một hoán vị của bộ $(4,1,0)$


#369135 Chứng minh hàm 2 biến số không liên tục : ..

Gửi bởi duongvanhehe trong 13-11-2012 - 10:37

Với từng biến thì làm như thế nào ạ?

Với từng biến thì theo mình chính là đi CM $\lim_{x\to 0}f(x;0)=0$ và $\lim_{y\to 0}f(0;y)=0$
Mà cái này thì hiển nhiên rồi (~~)


#369110 $\frac{a}{b}+\frac{b}{c...

Gửi bởi duongvanhehe trong 13-11-2012 - 00:20

Cho $a,b,c$ là các số dương thỏa mãn: $\frac{a}{b}+\frac{b}{c}+\frac{c}{a}=a+b+c$ .CMR:
$abc\geq 1$


#369108 Chứng minh hàm 2 biến số không liên tục : ..

Gửi bởi duongvanhehe trong 13-11-2012 - 00:01

Chứng minh rằng hàm:

\[z = \left\{ \begin{array}{l}
\frac{{2xy}}{{{x^2} + {y^2}}}\,\,\,khi\,\,{x^2} + {y^2} \ne 0 \\
0\,\,khi\,x = y = 0 \\
\end{array} \right.\]
liên tục trên mỗi biến riêng biệt, nhưng không liên tục đối với cả 2 biến tại $(0;0)$.

Với từng biến thì đơn giản rồi.Xét tính liên tục tại điểm ( $0;0$ ) :
Dãy điểm $(\frac{1}{n};\frac{1}{n})\rightarrow (0;0)$ khi $n\rightarrow \infty$
$\lim_{n\to +\infty}\frac{2.\frac{1}{n}.\frac{1}{n}}{\frac{1}{n^2}+\frac{1}{n^2}}=1\neq f(0;0)$
nên hàm số không liên tục tại điểm $(0;0)$


#369106 $$\frac{a}{a+bc}+\frac{b}...

Gửi bởi duongvanhehe trong 12-11-2012 - 23:42

Bài toán 1.
Ch0 các số thực dương $a,b,c$ thỏa $a+b+c=3$.Chứng minh rằng:
$$\frac{a}{a+bc}+\frac{b}{b+ac}+\frac{c}{c+ab}\geq \frac{3}{2}$$
Bài toán 2.
Ch0 các số thực dương $a,b,c$ có tổng là 3.Chứng minh rằng:
a) $$\frac{ab}{c}+\frac{bc}{a}+\frac{ca}{b}+abc\geq 4$$

Bài 1:
Bdt tương đương với:
$3+\frac{bc}{a}+\frac{ac}{b}+\frac{ab}{c}\geq a^2+b^2+c^2+3abc$
Điều này hiển nhiên đúng vì:
$3(3+\frac{bc}{a}+\frac{ac}{b}+\frac{ab}{c})=(a+b+c)^2+(a+b+c)(\frac{bc}{a}+\frac{ca}{b}+\frac{ab}{c})$
$=\sum a^2+3(ab+bc+ca)+\sum a^2(\frac{b}{c}+\frac{c}{b})\geq 3(a^2+b^2+c^2+ab+bc+ca)$
$\geq 3\sum a^2+9\sqrt[3]{a^2b^2c^2}\geq 3(\sum a^2+abc)$

Bài 2:
a) :wacko:
Ta có:
$9(\frac{ab}{c}+\frac{bc}{a}+\frac{ac}{b}+abc)$
$=2\sum a\sum \frac{ab}{c}+9abc+3\sum \frac{ab}{c}$
$=2(ab+bc+ca)+2\sum a^2(\frac{b}{c}+\frac{c}{b})+3\left ( \frac{ab}{c}+abc+\frac{ca}{b}+abc+\frac{bc}{a}+abc \right )$
$\geq 4\sum a^2+8(ab+bc+ca)=4(a+b+c)^2=36$
$\Rightarrow \frac{ab}{c}+\frac{bc}{a}+\frac{ac}{b}+abc\geq 4$


#366307 $$\frac{a^2+b}{a+b^2}+\frac{b^2+...

Gửi bởi duongvanhehe trong 01-11-2012 - 13:12

Thành quả sau 1 hồi lượm lặt :))
Bài toán 1.
Ch0 các số thực dương $a,b,c$ thỏa mãn $ab+bc+ca=3$.Chứng minh rằng:
$$\sqrt{a+3}+\sqrt{b+3}+\sqrt{c+3}\geq 6$$

Trước hết ta sẽ đi chứng minh với $\sqrt{a+3}+\sqrt{b+3}+\sqrt{c+3}=6$ thì $ab+bc+ca\leq 3$
Đặt $x=\frac{\sqrt{a+3}}{2};y=\frac{\sqrt{b+3}}{2};z=\frac{\sqrt{c+3}}{2}$ thì $x+y+z=3$ và ta sẽ phải CM
$6(x^2+y^2+z^2)\geq 6+4(x^2y^2+y^2z^2+z^2x^2)$
$\Leftrightarrow 6(x+y+z)^2-12(xy+yz+zx)\geq 6+4(xy+yz+zx)^{2}-8xyz(x+y+z)$
$\Leftrightarrow 12+6r\geq q^2+3q$
BĐT Schur $p^3+9r\geq 4pq\Leftrightarrow 3r\geq 4q-9$
Do đó ta cần CM $(q-2)(q-3)\leq 0$ .Do $q\leq 3$ nên ta sẽ có điều này khi $q\geq 2$
Trường hợp ngược lại $q\leq 2$ rõ ràng $q^2+3q\leq 10\leq 12+6r$
Túm lại là với $\sqrt{a+3}+\sqrt{b+3}+\sqrt{c+3}=6$ thì $ab+bc+ca\leq 3$
Lại đặt $m=ta;n=tb;p=tc$ sao cho $mn+np+pm=3\Rightarrow t\geq 1$
$\Rightarrow \frac{\sqrt{m+3}}{2}+\frac{\sqrt{n+3}}{2}+\frac{\sqrt{p+3}}{2}$
$\geq \frac{\sqrt{a+3}}{2}+\frac{\sqrt{b+3}}{2}+\frac{\sqrt{c+3}}{2}=6$ (do $t\geq 1)
Đây chính là điều phải CM
:))


#366008 $$a^3+b^3+c^3+5abc+4\geq 4(ab+bc+ca)$$

Gửi bởi duongvanhehe trong 30-10-2012 - 20:22

Bài toán 2.
Ch0 các số thực không âm $a,b,c$.Chứng minh bất đẳng thức sau:
$$a^3+b^3+c^3+5abc+4\geq 4(ab+bc+ca)$$

Sử dụng BĐT Schur và kết quả quen thuộc : $(a+b)(b+c)(c+a)\geq \frac{8}{9}(a+b+c)(ab+bc+ca)$
Ta có :
$a^3+b^3+c^3+5abc+4\geq (a+b)(b+c)(c+a)+4\geq \frac{8}{9}(a+b+c)(ab+bc+ca)+4$
$\geq \frac{8}{3\sqrt{3}}\sqrt{(ab+bc+ca)^3}+4$
$=4\left ( \frac{1}{3\sqrt{3}}\sqrt{(ab+bc+ca)^3}+ \frac{1}{3\sqrt{3}}\sqrt{(ab+bc+ca)^3}+1 \right )$
$\geq 4(ab+bc+ca)$ (AM-GM)
Đẳng thức xảy ra khi $a=b=c=1$


#365878 $\frac{a\left ( b+c \right )}{b^{2}+bc+c^{2}}+\frac{...

Gửi bởi duongvanhehe trong 30-10-2012 - 09:19

Anh Huyện có thể chữa bài này được không ạ? Em khá là bối rối khi gặp bất đẳng thức chứa đại lượng $[(a-b)(b-c)(c-a)]^2$.Cũng như bài toán này vậy:
http://diendantoanho...d-a-b2sum-a3b3/
Em cảm ơn anh ạ! :)

Chém thử coi sao : :icon10:
Đặt $a^2b+b^2c+c^2a=m$ và $ab^2+bc^2+ca^2=n$ .BĐT cần chứng minh tương đương với:
$\frac{a(b+c)}{b^2+bc+c^2}+\frac{b(a+c)}{c^2+ca+a^2}+\frac{c(a+b)}{a^2+ab+b^2}\geq \frac{m}{n}+\frac{n}{m}$
$\Leftrightarrow (m+n)\left (\frac{a(b+c)}{b^2+bc+c^2}+\frac{b(a+c)}{c^2+ca+a^2}+\frac{c(a+b)}{a^2+ab+b^2} \right )\geq m+n+\frac{m^2}{n}+\frac{n^2}{m}$
Ta có : $(m+n)\left (\frac{a(b+c)}{b^2+bc+c^2}+\frac{b(a+c)}{c^2+ca+a^2}+\frac{c(a+b)}{a^2+ab+b^2} \right )$
$=\sum \frac{a(b+c)(a^2(b+c)+a(b^2+c^2)+bc(b+c))}{b^2+bc+c^2}$
$=\sum \left ( a^3+a^2(b+c)+abc+\frac{a^3bc+ab^2c^2-a^2bc(b+c)}{b^2+bc+c^2} \right )$
$=a^3+b^3+c^3+3abc+m+n+abc\sum\frac{(a-b)(a-c)}{b^2+bc+c^2}\geq a^3+b^3+c^3+3abc+m+n$
Do đó ta chỉ cần chứng minh :
$a^3+b^3+c^3+3abc\geq \frac{m^2}{n}+\frac{n^2}{m}$
$\Leftrightarrow a^3+b^3+c^3+3abc-m-n\geq (\frac{1}{m}+\frac{1}{n})(m-n)^2$
$\Leftrightarrow (a+b-c)(a-b)^2+c(a-c)(b-c)\geq (\frac{1}{\sum a^2b}+\sum\frac{1}{ab^2})(a-b)^2(b-c)^2(c-a)^2$
Giả sử $c$ =min{ $a,b,c$ }.Ta có: $c(a-c)(b-c)\geq 0$ .Ta phải đi chứng minh :
$(a+b-c)\geq \left ( \frac{1}{\sum a^2b}+\frac{1}{\sum ab^2} \right )(a-c)^2(b-c)^2$
Do $\frac{(a-c)^2(b-c)^2}{a+b-c}\leq \frac{ab^2(a-c)}{(a-c)+b}=\frac{ab^2}{1+\frac{b}{a-c}}$
$\leq \frac{ab^2}{1+\frac{b}{a}}=\frac{a^2b^2}{a+b}$
$\Rightarrow (a+b-c)\geq \frac{a+b}{a^2b^2}(a-c)^2(b-c)^2=\left ( \frac{1}{a^2b}+\frac{1}{ab^2} \right )(a-c)^2(b-c)^2$
$\geq \left ( \frac{1}{\sum a^2b}+\frac{1}{\sum ab^2} \right )(a-c)^2(b-c)^2$
Suy ra đpcm.Đẳng thức xảy ra khi $a=b=c$ hoặc $abc=0$. ~O)


#365824 $$\sum \frac{{5{b^3} - {a^3...

Gửi bởi duongvanhehe trong 29-10-2012 - 21:40

Cho $a,b,c, > 0$
Chứng minh rằng:
$$\frac{{5{b^3} - {a^3}}}{{ab + 3{b^2}}} + \frac{{5{c^3} - {b^3}}}{{bc + 3{c^2}}} + \frac{{5{a^3} - {c^3}}}{{ac + 3{a^2}}} \leq a + b + c$$
Đây chính là đề thi HSG lớp 9 quận cầu giấy2005

Ta có : $\frac{5b^3-a^3}{ab+3b^2}+a-2b=\frac{5b^3-a^3+a^2b+3ab^2-2ab^2-6b^3}{ab+3b^2}$
$=\frac{-a^3-b^3+a^2b+ab^2}{ab+3b^2}=-\frac{(a+b)(a-b)^2}{ab+3b^2}\leq 0$
$\Rightarrow \frac{5b^3-a^3}{ab+3b^2}\leq 2b-a$
Tương tự ta có : $\frac{5c^3-b^3}{bc+3c^2}\leq 2c-b$ và $\frac{5a^3-c^3}{ca+3a^2}\leq 2a-c$
Cộng các BĐT trên llaij ta có: $\frac{{5{b^3} - {a^3}}}{{ab + 3{b^2}}} + \frac{{5{c^3} - {b^3}}}{{bc + 3{c^2}}} + \frac{{5{a^3} - {c^3}}}{{ac + 3{a^2}}} \leq a + b + c$
:icon6:


#365817 $\frac{1}{(a+b+c)^{2}}+\sum...

Gửi bởi duongvanhehe trong 29-10-2012 - 21:17

Bài toán:
Cho $a,b,c$ là các số thực không âm.Chứng minh rằng:
$\frac{1}{(a+b+c)^{2}}+\frac{1}{(a+b)^{2}}+\frac{1}{(b+c)^{2}}+\frac{1}{(c+a)^{2}}\geq \frac{5}{2(ab+bc+ca)}$

Mình xin trình bày 1 lời giải khác sử dụng BĐT Iran 96
Ta đã biết : $\frac{1}{(a+b)^2}+\frac{1}{(b+c)^2}+\frac{1}{(c+a)^2}\geq \frac{9}{4(ab+bc+ca)}$
Như vậy BĐT sẽ đúng nếu $\frac{1}{(a+b+c)^2}\geq \frac{1}{4(ab+bc+ca)}$
$\Leftrightarrow a^2+b^2+c^2\leq 2(ab+bc+ca)$
Xét trường hợp ngược lại : $a^2+b^2+c^2\geq 2(ab+bc+ca)$ .Ta có:
BĐT $\Leftrightarrow \frac{ab+bc+ca}{(a+b+c)^2}+\sum \frac{ab+bc+ca}{(a+b)^2}\geq \frac{5}{2}$
$\Leftrightarrow \frac{ab+bc+ca}{(a+b+c)^2}+\sum \frac{c}{a+b}+\sum \frac{ab}{(a+b)^2}\geq \frac{5}{2}$
Ta có : $\sum \frac{c}{a+b}+\sum \frac{ab}{(a+b)^2}\geq \frac{(a+b+c)^{2}}{2(ab+bc+ca)}+\sum \frac{ab}{2(a^2+b^2)}$
$= \frac{(a+b+c)^{2}}{2(ab+bc+ca)}+\frac{1}{4}\sum \frac{(a+b)^2}{a^2+b^2}-\frac{3}{4}$
$\geq \frac{(a+b+c)^{2}}{2(ab+bc+ca)}+\frac{(a+b+c)^{2}}{2(a^2+b^2+c^2)}-\frac{3}{4}$
Ta phải đi chứng minh:
$\frac{ab+bc+ca}{(a+b+c)^2}+\frac{(a+b+c)^2}{2(ab+bc+ca)}+\frac{(a+b+c)^2}{2(a^2+b^2+c^2)}-\frac{3}{4}\geq \frac{5}{2}$
Đặt $t=\frac{a^2+b^2+c^2}{ab+bc+ca}\geq 2$ thì BĐT trên tương đương với
$\frac{1}{2+t}+\frac{t}{2}+\frac{1}{t}\geq \frac{7}{4}\Leftrightarrow (t-2)(2t^2+t-4)\geq 0$
(Luôn đúng do $t \geq 2$)
CM hoàn tất.Đẳng thức xảy ra khi $a=b$ , $c=0$ hoặc các hoán vị.